Difference between revisions of "2016 AMC 10B Problems/Problem 16"
| m (→Solution 2) |  (→See Also) | ||
| Line 94: | Line 94: | ||
| {{AMC10 box|year=2016|ab=B|num-b=15|num-a=17}} | {{AMC10 box|year=2016|ab=B|num-b=15|num-a=17}} | ||
| {{MAA Notice}} | {{MAA Notice}} | ||
| + | [[Category: Introductory Algebra Problems]] | ||
Latest revision as of 17:59, 18 October 2025
Contents
Problem
The sum of an infinite geometric series is a positive number  , and the second term in the series is
, and the second term in the series is  . What is the smallest possible value of
. What is the smallest possible value of  
 
Solution 1
The sum of an infinite geometric series is of the form:
![\[\begin{split} S & = \frac{a_1}{1-r}  \end{split}\]](http://latex.artofproblemsolving.com/a/6/9/a697cd5bfeed3bf0b41ac850488992117247afd4.png) where
where  is the first term and
 is the first term and  is the ratio whose absolute value is less than 1.
 is the ratio whose absolute value is less than 1.
We know that the second term is the first term multiplied by the ratio. 
In other words:
![\[\begin{split} a_1 \cdot r & = 1 \\ a_1 & = \frac{1}{r} \end{split}\]](http://latex.artofproblemsolving.com/d/d/1/dd15facf892da1c8aaca8cfe51e4945d9e0e7c49.png) 
Thus, the sum is the following:
![\[\begin{split} S & = \frac{\frac{1}{r}}{1-r} \\\\ S & =\frac{1}{r-r^2} \end{split}\]](http://latex.artofproblemsolving.com/6/3/2/632da0913e6a0176c85f01f4b34a0f6e860638c4.png) 
Since we want the minimum value of this expression, we want the maximum value for the denominator,  .
The maximum x-value of a quadratic with leading coefficient
.
The maximum x-value of a quadratic with leading coefficient  is
 is  .
.
![\[\begin{split} r & = \frac{-(1)}{2(-1)} \\\\ r & = \frac{1}{2}  \end{split}\]](http://latex.artofproblemsolving.com/5/2/a/52a3bcebe1757c74c9a058f36cab56296cc76818.png) 
Plugging  
  
  into the quadratic yields:
 into the quadratic yields:
![\[\begin{split} S & = \frac{1}{\frac{1}{2} -\left(\frac{1}{2}\right)^2} \\\\ S & = \frac{1}{\frac{1}{4}}  \end{split}\]](http://latex.artofproblemsolving.com/b/0/4/b04356d84bcc75bd9078cebba044ef9c8a5dcd24.png) 
Therefore, the minimum sum of our infinite geometric sequence is  .
(Solution by akaashp11)
.
(Solution by akaashp11)
As an extension to find the maximum value for the denominator we can find the derivative of  to get
 to get  . we know that this changes sign when
. we know that this changes sign when  so plugging it in into the original equation we find the answer is
 so plugging it in into the original equation we find the answer is  .
.
Solution 2
After observation we realize that in order to minimize our sum  with
 with  being the reciprocal of r, the common ratio
 being the reciprocal of r, the common ratio  has to be in the form of
 has to be in the form of  , with
, with  being an integer, as anything more than
 being an integer, as anything more than  divided by
 divided by  would give a larger sum than a ratio in the form of
 would give a larger sum than a ratio in the form of  .
. 
The first term has to be  , so then in order to minimize the sum, we have minimize
, so then in order to minimize the sum, we have minimize  .
. 
The smallest possible value for  such that it is an integer that's greater than
 such that it is an integer that's greater than  is
 is  . So our first term is
. So our first term is  and our common ratio is
 and our common ratio is  . Thus the sum is
. Thus the sum is  or
 or  .
Solution 2 by No_One
.
Solution 2 by No_One
(edited)
Solution 3
We can see that if  is the first term, and
 is the first term, and  is the common ratio between each of the terms, then we can get
 is the common ratio between each of the terms, then we can get
![\[S=\frac{a}{1-r} \implies S-Sr=a\]](http://latex.artofproblemsolving.com/a/d/b/adbaef963036b32da5c449fc3cb7297470b4779f.png) Also, we know that the second term can be expressed as
Also, we know that the second term can be expressed as  notice if we multiply
notice if we multiply  by
 by  , we would get
, we would get
![\[r(S-Sr)=ar \implies Sr-Sr^2=1 \implies Sr^2-Sr+1=0\]](http://latex.artofproblemsolving.com/1/0/7/10721f8f40778c79d481ab4ad623c7bcaad89ee3.png) This quadratic has real solutions if the discriminant is greater than or equal to zero, or
This quadratic has real solutions if the discriminant is greater than or equal to zero, or
![\[S^2-4\cdot S \cdot 1 \ge 0\]](http://latex.artofproblemsolving.com/8/3/3/8335bbaac48d9e07285e50a19806030cfbfacd89.png) This yields that
This yields that  or
 or  . 
However, since we know that
. 
However, since we know that  has to be positive, we can safely conclude that the minimum possible value of
 has to be positive, we can safely conclude that the minimum possible value of  is
 is  .
.
Solution 4 (Quick Method)
Let the first term of the geometric series  . Since it must be decreasing, we have
. Since it must be decreasing, we have  and the third term is
 and the third term is  . Realize that by AM-GM inequality
. Realize that by AM-GM inequality  with equality if
 with equality if  . However, we established that
. However, we established that  so that means
 so that means  . So the sum of the first three terms of the sequence
. So the sum of the first three terms of the sequence  is greater than
 is greater than  , and the geometric series keeps continuing  infinitely.  This means the sum continues increasing. The only answer choice greater than
, and the geometric series keeps continuing  infinitely.  This means the sum continues increasing. The only answer choice greater than  is
 is  . ~skyscraper
. ~skyscraper
Solution 5 (Clever Algebra)
Let the first term be  The sum of the series is
 The sum of the series is  Rewrite this as
 Rewrite this as  By AM-GM we know that
 By AM-GM we know that  so the minimum is
 so the minimum is  
Solution 6 (Calculus)
Set the first term is  , the common ratio should be
, the common ratio should be  
 
The sum to infinity of the series is  
Since  is positive, we have
 is positive, we have  Define the function
  Define the function  , the domain of this function is
 , the domain of this function is  
Let  We solve that
 We solve that  
It's easy to find that when  when
 when  Thus
 Thus  has a minimum value when
 has a minimum value when  , which is
, which is  Choose
 Choose   
~PythZhou
See Also
| 2016 AMC 10B (Problems • Answer Key • Resources) | ||
| Preceded by Problem 15 | Followed by Problem 17 | |
| 1 • 2 • 3 • 4 • 5 • 6 • 7 • 8 • 9 • 10 • 11 • 12 • 13 • 14 • 15 • 16 • 17 • 18 • 19 • 20 • 21 • 22 • 23 • 24 • 25 | ||
| All AMC 10 Problems and Solutions | ||
These problems are copyrighted © by the Mathematical Association of America, as part of the American Mathematics Competitions.  
